Email
Chat with tutors
logo

Ask Questions, Get Answers

X
 
Questions  >>  CBSE XII  >>  Math  >>  Matrices
Answer
Comment
Share
Q)

For what value of '‘K'’ the matrix $ \begin{bmatrix} k & 2 \\ 3 & 4 \end{bmatrix}$has no inverse.

Please log in or register to answer this question.

Home Ask Tuition Questions
Your payment for is successful.
Continue
...